-
Notifications
You must be signed in to change notification settings - Fork 2
/
Copy pathMulti_Continuity.tex
315 lines (238 loc) · 10.1 KB
/
Multi_Continuity.tex
1
2
3
4
5
6
7
8
9
10
11
12
13
14
15
16
17
18
19
20
21
22
23
24
25
26
27
28
29
30
31
32
33
34
35
36
37
38
39
40
41
42
43
44
45
46
47
48
49
50
51
52
53
54
55
56
57
58
59
60
61
62
63
64
65
66
67
68
69
70
71
72
73
74
75
76
77
78
79
80
81
82
83
84
85
86
87
88
89
90
91
92
93
94
95
96
97
98
99
100
101
102
103
104
105
106
107
108
109
110
111
112
113
114
115
116
117
118
119
120
121
122
123
124
125
126
127
128
129
130
131
132
133
134
135
136
137
138
139
140
141
142
143
144
145
146
147
148
149
150
151
152
153
154
155
156
157
158
159
160
161
162
163
164
165
166
167
168
169
170
171
172
173
174
175
176
177
178
179
180
181
182
183
184
185
186
187
188
189
190
191
192
193
194
195
196
197
198
199
200
201
202
203
204
205
206
207
208
209
210
211
212
213
214
215
216
217
218
219
220
221
222
223
224
225
226
227
228
229
230
231
232
233
234
235
236
237
238
239
240
241
242
243
244
245
246
247
248
249
250
251
252
253
254
255
256
257
258
259
260
261
262
263
264
265
266
267
268
269
270
271
272
273
274
275
276
277
278
279
280
281
282
283
284
285
286
287
288
289
290
291
292
293
294
295
296
297
298
299
300
301
302
303
304
305
306
307
308
309
310
311
312
\chapter{多元极限与连续}
\section{累次极限与重极限}
\subsection{基本概念}
\begin{definition}[重极限]
$a \in \mathbb{R}^n$,$f: \mathbb{R}^n \rightarrow \mathbb{R}$在$a$的去心邻域中有定义,
若$\exists A, \forall \epsilon, \exists \delta$当$\mathbf{x} \in \mathbb{R}^n$满足$|\mathbf{x} - a| < \delta$时,
有$|f(\mathbf{x}) - A| < \epsilon$,
则称$\lim \limits _{\mathbf{x} \rightarrow a}f(\mathbf{x}) = A$
\end{definition}
\begin{note}
计算重极限的方法:
\begin{itemize}
\item 放缩+迫敛:例如$\left| (x+y) \sin \frac{1}{x } \sin \frac{1}{y} \right| \leq |x+y| = 0$
\item 极坐标变换:例如$\left| \frac{x^3 + y^3}{x^2 + y^2} \right| = \left| \frac{r^3\cos^3\theta + r^3 \sin^3 \theta}{r^2} \right| = r|\cos^3 \theta + \sin^3 \theta| \leq 2r \rightarrow 0$
\end{itemize}
\end{note}
\begin{note}
证明重极限不存在的常用方法:
\begin{itemize}
\item 两个累次极限存在但不相等
\item 找特殊的趋近方式,如$y = kx$,使得获得的重极限不同
\end{itemize}
\end{note}
~
\begin{exercise}[重极限的计算]
(1)已知$f(x,y)$如下,计算$\lim \limits _{(x,y) \rightarrow (0,0)}f(x,y)$
\begin{equation*}
f(x,y) =
\begin{cases}
xy \frac{x^2 - y^2}{x^2 + y^2}, &(x,y) \neq 0\\
0, &(x,y) = 0
\end{cases}
\end{equation*}
\end{exercise}
\begin{solution}
(1)设$x = r \cos \theta, y = r \sin \theta$,
则
\begin{equation*}
|f(x,y)| = \left|\frac{r^2 \sin \theta \cos \theta (r^2\cos ^2\theta - r^2\sin^2 \theta)}{r^2}\right| = \frac{1}{4}r^2 \left| \sin 4\theta \right| \leq \frac{1}{4}r^2\rightarrow 0
\end{equation*}
因此极限为$0$
\end{solution}
~
\begin{definition}[累次极限]
设$f(x,y)$在$x_0,y_0$的某去心邻域中有定义,
则称$\lim \limits _{y \rightarrow y_0}\lim \limits _{x \rightarrow x_0} f(x,y), \lim \limits _{x \rightarrow x_0} \lim \limits _{y \rightarrow y_0}f(x,y)$称为累次极限。
\end{definition}
\begin{note}
重极限是多个分量同时趋近于$\mathbf{x}_0$,
而累次极限相当于依次取极限的极限。
\end{note}
\begin{theorem}[重极限与累次极限的关系]
若重极限$\lim \limits _{(x,y) \rightarrow (x_0,y_0)}f(x,y) = A$存在,且累次极限存在(累次极限可能不存在),
则累次极限的值为$A$
\end{theorem}
\begin{proof}
$\forall \epsilon, \exists \delta, |\mathbf{x} - \mathbf{x}_0| < \delta$时,
$|f(x,y) - A| < \epsilon$,
两侧取极限$x \rightarrow x_0$,
得到$|\lim \limits _{x \rightarrow x_0}f(x,y) - A| < \epsilon$,
当$|y - y_0| < \delta$时显然结论成立。
\end{proof}
\begin{theorem}[重极限的任意性]
设$(x,y)$沿着任意路径趋于$(x_0,y_0)$时,$f(x,y) \rightarrow A$,则
\begin{equation*}
\lim \limits _{(x,y) \rightarrow (x_0,y_0)} f(x,y) = A
\end{equation*}
\end{theorem}
\begin{proof}
反设$\lim \limits _{(x,y) \rightarrow (x_0,y_0)}f(x,y) \neq A$,
则$\exists \epsilon_0$和$\{\mathbf{x}_n\}$使得$\lim \limits _{n \rightarrow \infty} \mathbf{x}_n = (x_0,y_0)$,
但$|f(\mathbf{x}_n) - A| \geq \epsilon_0$,
将$\mathbf{x}_1,\mathbf{x}_2,\cdots,\mathbf{x}_n,\cdots$连接成折线,
折线趋近于$(x_0,y_0)$,但$f(x,y)$不趋于$A$,这与已知矛盾。
\end{proof}
~
\begin{exercise}[常用反例]
讨论$(0,0)$处的重极限与累次极限
(1)重点:$f(x,y) = \frac{xy}{x^2 + y^2}$
(2)$f(x,y) = \frac{x-y}{x+y}$
(3)$f(x,y) = x \sin \frac{1}{y}$
(4)$f(x,y) = x \sin \frac{1}{y} + y \sin \frac{1}{x}$
(5)重点:$f(x,y) =
\begin{cases}
1, &0 < y<x^2 , -\infty < x <+\infty\\
0, &\text{其他}
\end{cases}
$
\end{exercise}
\begin{solution}
(1)累次都是$0$。
重极限若选择$y = kx$,
则得到$f(x,y) = \frac{kx^2}{(k^2 + 1)x^2} = \frac{k}{k^2 + 1}$,
重极限不存在。
(2)累次为$\pm 1$。
重极限考虑$y = kx$,则$f(x,y) = \frac{x - kx}{x + kx} = \frac{1 - k}{1 + k}$,
不存在
(3)先$x$后$y$极限为$0$,
先$y$后$x$不存在。
重极限$x \sin \frac{1}{y} < |x| \rightarrow 0$,
因此重极限为$0$
(4)累次极限不存在。
而$|f(x,y)| \leq |x| + |y| \rightarrow 0$,
重极限为$0$。
(5)画出图像,发现沿着直线逼近都是$0$,沿着抛物线$y = kx^2(0 < k < 1)$时趋于$1$,因此极限不存在。
\end{solution}
~
\begin{exercise}[反例应用]
上述反例可作为以下问题的解答:
\begin{enumerate}
\item 累次极限存在但不相等:$\frac{x-y}{x+y}$
\item 累次极限存在且相等,但重极限不存在:$\frac{xy}{x^2 + y^2}$
\item 累次极限不存在,但重极限存在:$x \sin \frac{1}{y} + y \sin \frac{1}{x}$
\item 重极限与双累次极限都不存在:$\frac{1}{x} + \frac{1}{y}$
\end{enumerate}
\end{exercise}
~
\subsection{重极限的计算}
~
\begin{exercise}[多元分式]
讨论$(0,0)$处的重极限:
(1)$f(x,y) = \frac{xy}{x+y}$
(2)$f(x,y) = \frac{xy}{x^2 + y^2}$
(3)$f(x,y) = \frac{x^2y^2}{(x^2 + y^2)^2}$
(4)$f(x,y) = \frac{x^6y^8}{(x^2 + y^4)^5}$
(5)$f(x,y) = \frac{x^2y^2}{x^2y^2 + (x-y)^2}$
(6)$f(x,y) = \frac{x^2y^2}{x^3 + y^3}$
\end{exercise}
\begin{solution}
(1)不齐次,一方面将分子次数升高,$y = x$得到极限为$0$。
另一方面常数消去公因式,将分母变简单,例如$y = x^2 - x$化简为$x - 1 \rightarrow -1$,因此重极限不存在
(2)齐次,$y = kx$,不存在
(3)齐次,$y = kx$,不存在
(4)尝试将分母变齐次,
$x = ky^2$,得到$\frac{k^6y^{20}}{(k^2+1)^5y^{20}}$,不存在
(5)尝试化简$x = y$得到$1$,
其他想不到办法,尝试取零,$x = 0$得到极限为$0$。不存在
(6)分子次数高:$y = x$显然极限为$0$。
另一方面化简分母,设$y^3 = x^4 - x^3$,
得到$\frac{x^2(x^4x^3)^{\frac{2}{3}}}{x^4}$,
极限不是$0$。
\end{solution}
~
\begin{exercise}[指数形式]
(1)$\lim \limits _{(x,y) \rightarrow (0^+,0^+)}x^y$
\end{exercise}
\begin{solution}
(1)用累次极限看出一个$0$,一个$1$,显然重极限不存在
\end{solution}
\section{多元函数连续性}
\subsection{多元函数连续性}
\begin{definition}[多元函数连续]
若$f(x,y)$在$(x_0,y_0)$处满足$\lim \limits _{(x,y) \rightarrow (x_0,y_0)} f(x,y) = f(x_0,y_0)$,
则称$f(x,y)$在$(x_0,y_0)$处连续。
\end{definition}
\begin{example}[关于$x,y$连续,不多元连续]
举例说明$f(x,y)$关于$x,y$均连续,
但不多元连续。
\end{example}
\begin{solution}
不一定,例如:
\begin{equation*}
f(x,y) =
\begin{cases}
\frac{xy}{x^2 + y^2}, & (x,y) \neq (0,0)\\
0, & (x,y) = (0,0)
\end{cases}
\end{equation*}
$f(x,y)$关于$x,y$均连续,
但是$\lim \limits _{(x,y) \rightarrow (0,0)}f(x,y)$不存在。
\end{solution}
~
\begin{exercise}[加强条件]
若$f(x,y)$在$D$中关于$x,y$连续,且增加任意条件可证明$f(x,y)$连续:
\begin{enumerate}
\item 一致连续条件:$y$的连续是关于$x$一致的
\item Lip条件:$f(x,y)$关于$x$满足Lip条件,则$f(x,y)$连续
\item 单调条件:关于$x$或$y$单调
\end{enumerate}
\end{exercise}
\begin{proof}
(1)(2)用$|f(x,y) - f(x_0,y_0)| \leq |f(x,y) - f(x,y_0)| + |f(x,y_0) - f(x_0,y_0)| < 2\epsilon$即可,
注意第一个绝对值放缩要求$y$的连续关于$x$一致!
(3)设关于$y$单调增。
考虑一点$(x_0,y_0)$,
用$(x,y)$逼近,
设$y_0 - \delta_1 < y < y_0 + \delta_1$,
首先根据$y$的单调性可知$f(x,y) \leq f(x,y_0 + \delta_1)$
根据$y$的连续性可知$f(x,y_0 + \delta_1) \leq f(x, y_0) + \epsilon$。
进一步地根据$x$的连续性可知
\begin{equation*}
f(x,y) \leq f(x,y_0) + \epsilon \leq f(x_0,y_0) + 2\epsilon
\end{equation*}
同理可证得$f(x,y) \geq f(x_0,y_0) - 2\epsilon$,
综上可证$|f(x,y) - f(x_0,y_0)| \leq 2\epsilon$。
\end{proof}
\subsection{多元函数的一致连续性}
\begin{lemma}[多元三角不等式]
对$\forall \mathbf{x}, \mathbf{y} \in \mathbb{R}^n$,
有$\left| |\mathbf{x}| - |\mathbf{y}| \right| \leq |\mathbf{x} - \mathbf{y}|$
\end{lemma}
\begin{proof}
$|\mathbf{x}| = |(\mathbf{x}-\mathbf{y}) + \mathbf{y}| \leq |\mathbf{x} - \mathbf{y}| + |\mathbf{y}|$,
因此移项得到$|\mathbf{x}| - |\mathbf{y}| \leq |\mathbf{x} - \mathbf{y}|$。
同理可得$|\mathbf{y}| - |\mathbf{x}| \leq |\mathbf{x} - \mathbf{y}|$
\end{proof}
~
\begin{exercise}[多元一致连续]
(1)$f(\mathbf{x}) = |\mathbf{x}|$,证明$f(\mathbf{x})$在$\mathbb{R}^n$上一致连续
(2)$f(x,y) = \sin(xy)$在$\mathbb{R}^2$的一致连续性
(3)$f(x,y) = \frac{1}{1 - xy}$在$(0,1) \times (0,1)$的一致连续性
\end{exercise}
\begin{proof}
(1)$\forall \epsilon, \forall \mathbf{x},\mathbf{y} \in \mathbb{R}^n$,
若$|\mathbf{x} - \mathbf{y}| < \epsilon$,
则根据三角不等式可知$|f(\mathbf{x}) - f(\mathbf{y})| = ||\mathbf{x}| - |\mathbf{y}|| \leq |\mathbf{x} - \mathbf{y}| < \epsilon$,
因此取$\delta = \epsilon$即可。
(2)选$a_n = (\sqrt{2n\pi}, \sqrt{2n\pi}), b_n = (\sqrt{2n\pi + \frac{\pi}{2}}, \sqrt{2n\pi + \frac{\pi}{2}})$,
得到$|b_n - a_n| \rightarrow 0$,
但$|f(b_n) - f(a_n)| \not \rightarrow 0$,因此不一致连续
(3)由于是在$(1,1)$处出问题,
可以考虑$a_n = (1 - \frac{1}{n}, 1 - \frac{1}{n}), b_n = (1 - \frac{1}{n+1}, 1 - \frac{1}{n+1})$,
显然$|a_n - b_n| \rightarrow 0$,
$|f(a_n) - f(b_n)| = \left| \frac{n^2 - 3n^2 + o(n^2)}{4n^2 - 1} \right| \rightarrow \frac{1}{2}$,
因此不一致连续。
\end{proof}
~
\begin{theorem}[多元Lip连续与一致连续]
$f(x,y)$是二元函数,若对$\forall (x_1,y),(x_2,y),(x,y_1),(x,y_2) \in \mathbb{R}^2$有
\begin{equation*}
|f(x_1,y) - f(x_2,y)| \leq L|x_1 - x_2|, \quad |f(x,y_1) - f(x,y_2)| \leq L|y_1 - y_2|
\end{equation*}
则$f(x,y)$在$\mathbb{R}^2$上一致连续
\end{theorem}
\begin{proof}
考虑$|f(x^{\prime},y^{\prime}) - f(x^{\prime\prime},y^{\prime\prime})| \leq |f(x^{\prime},y^{\prime}) - f(x^{\prime},y^{\prime\prime})| + |f(x^{\prime},y^{\prime\prime}) - f(x^{\prime\prime},y^{\prime\prime})|$,
用Lip条件放缩即可。
\end{proof}